Linear transformation of an orthonormal basis

Click For Summary
SUMMARY

The discussion centers on demonstrating that a linear transformation L from Rm to Rn can produce an orthogonal set of vectors from an orthonormal basis {v1,...,vm} of Rm. The key insight is to utilize the symmetric matrix ATA, which is diagonalizable by orthogonal matrices, ensuring the existence of an orthonormal basis. The conclusion is that the transformed vectors {L(v1),...,L(vm)} are orthogonal, with the possibility of some being zero.

PREREQUISITES
  • Understanding of linear transformations in vector spaces
  • Knowledge of symmetric matrices and their properties
  • Familiarity with orthonormal bases and eigenvectors
  • Concept of diagonalization of matrices
NEXT STEPS
  • Study the properties of symmetric matrices and their eigenvalues
  • Learn about the process of diagonalization using orthogonal matrices
  • Explore the implications of linear transformations on vector orthogonality
  • Investigate the relationship between orthonormal bases and linear transformations
USEFUL FOR

Mathematics students, particularly those studying linear algebra, educators teaching vector space concepts, and researchers focusing on linear transformations and their applications.

zwingtip
Messages
20
Reaction score
0

Homework Statement


Consider a linear transformation L from Rm to Rn. Show that there is an orthonormal basis {v1,...,vm} of Rm to Rn such that the vectors {L(v1),...,L(vm)} are orthogonal. Note that some of the vectors L(vi) may be zero. HINT: Consider an orthonormal basis {v1,...,vm} for the symmetric matrix ATA.


Homework Equations


if v1 and v2 are eigenvectors of a symmetric matrix with distinct eigenvalues \lambda_1 and \lambda_2, then v1 and v2 are orthogonal


The Attempt at a Solution


I have no idea how to even start this problem and I've been trying for a couple of days. Can anybody give me a tip as to how attack it? Thanks.
 
Physics news on Phys.org
The matrix A^T A is diagonalizable by orthogonal matrices since it is symmetric. Therefore there exists an orthonormal basis v_1,..., v_m such that A^T A v_i = c_i v_i for some constant c_i. Now can you show that the Av_i are all orthogonal to each other?
 
Question: A clock's minute hand has length 4 and its hour hand has length 3. What is the distance between the tips at the moment when it is increasing most rapidly?(Putnam Exam Question) Answer: Making assumption that both the hands moves at constant angular velocities, the answer is ## \sqrt{7} .## But don't you think this assumption is somewhat doubtful and wrong?

Similar threads

Replies
7
Views
2K
  • · Replies 11 ·
Replies
11
Views
3K
Replies
8
Views
3K
  • · Replies 3 ·
Replies
3
Views
8K
  • · Replies 2 ·
Replies
2
Views
2K
  • · Replies 14 ·
Replies
14
Views
4K
  • · Replies 3 ·
Replies
3
Views
5K
  • · Replies 1 ·
Replies
1
Views
2K
  • · Replies 4 ·
Replies
4
Views
5K
  • · Replies 2 ·
Replies
2
Views
2K